Đến nội dung

Hình ảnh

Chuyên đề Phần nguyên


  • Please log in to reply
Chủ đề này có 42 trả lời

#1
Zaraki

Zaraki

    PQT

  • Phó Quản lý Toán Cao cấp
  • 4273 Bài viết
PHẦN NGUYÊN CỦA MỘT SỐ THỰC


A. ĐỊNH NGHĨA

Ta biết rằng, mọi số thực $x$ đều có thể viết được dưới dạng

$x=n+z$

trong đó $n$ là số nguyên và $0 \le z \le 1$

Chẳng hạn:

$7,9=7+0,9$
$-7,9=-8+0,1$

Hơn nữa, cách viết như trên là duy nhất. Ta gọi số nguyên $n$ là phần nguyên của $x$ và kí hiệu là $[x]$; còn $z$ được gọi là phần phân của $x$ và kia hiệu là $\left \{ x \right \}$.

Từ phân tích, ta rút ra định nghĩa
Định nghĩa: Phần nguyên của số thực $x$, kí hiệu là $[x]$, là số nguyên lớn nhất không vượt quá $x$. Phần phân của số thực $x$ được định nghĩa bởi $\left \{ x \right \}=x-[x]$.

Ngoài cách gọi thông thường là phần nguyên (intergal part) của $x$ với kí hiệu là $[x]$, một số tác giả nước ngoài còn gọi đó là floor function và kí hiệu là $\left \lfloor x \right \rfloor$. Sở dĩ thế vì người ta nêu ro ceiling function - kí hiệu $\left \lceil x \right \rceil$, như định nghĩa sau đây

$\left \lceil x \right \rceil$ là số nguyên nhỏ nhất vượt quá $x$


Dễ dàng thấy rằng

$\left \lceil x \right \rceil=\left\{\begin{matrix}x=\left \lfloor x \right \rfloor ; x\in \mathbb{Z} & & \\ \left \lceil x \right \rceil+1 ; x\notin \mathbb{Z} & & \end{matrix}\right.$


B. TÍNH CHẤT

1) $x=[x]+\left \{ x \right \}$
2) $x=[x] \Leftrightarrow x \in \mathbb{Z}$
3) $x=\left \{ x \right \} \Leftrightarrow 0\le x < 1$
4) $x-1<[x] \le x$
5) Nếu $k$ nguyên thì


$[x+k]=[x]+k$ và $\left \{ x+k \right \}=\left \{ x \right \}+k$

Bạn hãy tập chứng minh những tính chất này đi!

Xin đưa thêm một số tính chất
6) $[x+y] \ge [x]+[y]$
7) $[x] \le x <[x]+1$
8) Nếu $x \ge y$ thì $[x] \ge [y]$

9) $0 \le \left \{ x \right \} <1$
10) $\left \{ x+y \right \} \le \left \{ x \right \} + \left \{ y \right \}$

Chứng minh tính chất 6
Viết $x=[x]+\left \{ x \right \}, y=[y]+\left \{ y \right \}$
Khi đó
$[x+y]=[([x]+[y])+(\left \{ x \right \}+\left \{ y \right \})]=[x]+[y]+[\left \{ x \right \}+\left \{ y \right \}]$. (1)
Vì $\left \{ x \right \} \ge 0$ và $\left \{ y \right \} \ge 0$ nên $[\left \{ x \right \}+\left \{ y \right \}] \ge 0$.
Kết hợp với (1) ta suy ra

$[x+y] \ge [x]+[y]$


Chứng minh tính chất 8
Vì $x \ge y$ nên $\exists \alpha \ge 0$ sao cho:

$x=y+\alpha$ hay $x=[y]+(\left \{ y \right \}+\alpha)$.


Suy ra $[x]=[y]+[(\left \{ y \right \}+\alpha)]$. (1)
Vì $\alpha \ge 0$ và $\left \{ y \right \} \ge 0$ nên $\left \{ y \right \}+\alpha \ge 0$ và $[(\left \{ y \right \}+ \alpha)] \ge 0$.
Kết hợp với (1) ta có $[x] \ge [y]$.

Xin giới thiệu thêm một số tính chất khá là thú vị
1) Giả sử $0<\alpha \in \mathbb{R}$ và $n \in \mathbb{N}$. Lúc đó $\left [ \dfrac{\alpha}{n} \right ]$ là số tất cả các số nguyên dương là bội của $n$ nhưng không vượt quá $\alpha$.
2) Giả sử $0<\alpha \in \mathbb{R}$ và $n \in \mathbb{N}$. Lúc đó,

$\left [ \dfrac{n}{\alpha} \right ]$

là số tất cả các số nguyên dương là bội của $\alpha$ nhưng không vượt quá $n$.
3) Nếu $a$ và $b$ là hai số không âm, thì

$[2a]+[2b] \ge [a]+[b] + [a+b]$


Bài viết đã được chỉnh sửa nội dung bởi E. Galois: 19-08-2012 - 08:56

Discovery is a child’s privilege. I mean the small child, the child who is not afraid to be wrong, to look silly, to not be serious, and to act differently from everyone else. He is also not afraid that the things he is interested in are in bad taste or turn out to be different from his expectations, from what they should be, or rather he is not afraid of what they actually are. He ignores the silent and flawless consensus that is part of the air we breathe – the consensus of all the people who are, or are reputed to be, reasonable.

 

Grothendieck, Récoltes et Semailles (“Crops and Seeds”). 


#2
NguyThang khtn

NguyThang khtn

    Thượng úy

  • Hiệp sỹ
  • 1468 Bài viết
Đây là một số tài liệu của thầy hxthanh :
Chuyên đề phần nguyên

It is difficult to say what is impossible, for the dream of yesterday is the hope of today and the reality of tomorrow

 


#3
Zaraki

Zaraki

    PQT

  • Phó Quản lý Toán Cao cấp
  • 4273 Bài viết
Anh Thắng có thể post một số tính chất nữa về phần nguyên lên đây được không anh? Cảm ơn anh nhiều!

Discovery is a child’s privilege. I mean the small child, the child who is not afraid to be wrong, to look silly, to not be serious, and to act differently from everyone else. He is also not afraid that the things he is interested in are in bad taste or turn out to be different from his expectations, from what they should be, or rather he is not afraid of what they actually are. He ignores the silent and flawless consensus that is part of the air we breathe – the consensus of all the people who are, or are reputed to be, reasonable.

 

Grothendieck, Récoltes et Semailles (“Crops and Seeds”). 


#4
hxthanh

hxthanh

    Tín đồ $\sum$

  • Hiệp sỹ
  • 3915 Bài viết
Bạn Phạm Quang Toàn thân mến!

Tôi rất vui vì bạn cũng rất quan tâm đến chủ đề này
Phần Nguyên là một chủ đề rất khó, mà giáo trình chúng ta được học lại không có nhiều.
Trong rất nhiều bài toán số học, vô tình ta vẫn dùng đến kiến thức của hàm này mà trong cách suy luận mà không hiểu được nguồn gốc của nó.
Một chương trình giới thiệu đầy đủ về Phần Nguyên, hiện nay vẫn còn là một đề tài mở.
Vì vậy, tôi, bạn và tất cả chúng ta sẽ cùng nhau trao đổi những kiến thức đã biết, những bài toán đã trải nghiệm hay những gì mà các bạn đã nghiên cứu được về Phần Nguyên để tất cả mọi người tham khảo.

Dưới đây, tôi xin đưa ra một số bài tập:

1. Tìm giá trị lớn nhất và nhỏ nhất của
$P=\dfrac{2x\left\lfloor 3x \right\rfloor -x^2+4}{{\left\lfloor x \right\rfloor}^2+2x^2+1}$

2. Biểu diễn dãy số sau bằng một công thức duy nhất!
$1,1,2,3,4,9,8,27,...,2^{n-1},3^{n-1},...$

3. Chứng minh rằng với mọi n nguyên dương ta có:
$\left\lfloor\sqrt{n}+\dfrac{1}{2}\right\rfloor+\left\lfloor\sqrt{n-1}\right\rfloor=\left\lfloor\sqrt{4n-3}\right\rfloor$

Tạm thời thế đã :icon6:

Bài viết đã được chỉnh sửa nội dung bởi hxthanh: 02-11-2011 - 07:26


#5
Zaraki

Zaraki

    PQT

  • Phó Quản lý Toán Cao cấp
  • 4273 Bài viết
Đang băn khoăn một số bài, xin đưa lên

1. Tìm số tự nhiên $k$ lớn nhất sao cho
$$(1994!)^{1995} \ \vdots \ 1995^k$$
2. Có bao nhiêu số tự nhiên $x$ thỏa mãn
$$\left[ \dfrac{x}{2010} \right] = \left[ \dfrac{x}{2011} \right] = \left[ \dfrac{x}{2012} \right]$$
3. Tìm $x$ biết
$$ \left[ -x^2+3x \right] = \left[ x^2+ \dfrac{1}{2} \right]$$

4. Tìm $x$ biết
$$[x]^2-[x]-2=0$$

Bài viết đã được chỉnh sửa nội dung bởi Phạm Quang Toàn: 01-11-2011 - 20:04

Discovery is a child’s privilege. I mean the small child, the child who is not afraid to be wrong, to look silly, to not be serious, and to act differently from everyone else. He is also not afraid that the things he is interested in are in bad taste or turn out to be different from his expectations, from what they should be, or rather he is not afraid of what they actually are. He ignores the silent and flawless consensus that is part of the air we breathe – the consensus of all the people who are, or are reputed to be, reasonable.

 

Grothendieck, Récoltes et Semailles (“Crops and Seeds”). 


#6
hxthanh

hxthanh

    Tín đồ $\sum$

  • Hiệp sỹ
  • 3915 Bài viết
Bài 1: Sử dụng định lý Lagrande về số mũ của cao nhất của một số nguyên tố chứa trong n!

Ta có $1995=3.5.7.19$

Theo định lý Legendre thì số mũ cao nhất của $19$ có trong $(1994)!$ là:

$\left\lfloor\dfrac{1994}{19}\right\rfloor+\left\lfloor\dfrac{1994}{(19)^2}\right\rfloor+...+\left\lfloor\dfrac{1994}{(19)^k}\right\rfloor+...=109$

Như vậy $(1994)!\;\vdots\; (1995)^{109}$ và $(1994)!\;\not{\vdots} \;(1995)^{n\ge110}$

Suy ra để $((1994)!)^{1995}\;\vdots\; (1995)^k$ thì $k\le 109*1995=217\;455$

#7
hxthanh

hxthanh

    Tín đồ $\sum$

  • Hiệp sỹ
  • 3915 Bài viết
Bài 2

Xét hệ phương trình: $\left\lfloor\dfrac{x}{2010}\right\rfloor=\left\lfloor\dfrac{x}{2011}\right\rfloor=\left\lfloor\dfrac{x}{2012}\right\rfloor$

Vì $x\in\mathbb{N}$ nên ta có thể đặt $x=2010k+r,\;\;\;(0\le k\in\mathbb{N};\;0\le r \le 2009$

Thay vào hệ trên ta có:

$\left\lfloor\dfrac{2010k+r}{2010}\right\rfloor=\left\lfloor\dfrac{2011k+r-k}{2011}\right\rfloor=\left\lfloor\dfrac{2012k+r-2k}{2012}\right\rfloor$

$\Leftrightarrow k=k+\left\lfloor\dfrac{r-k}{2011}\right\rfloor=k+\left\lfloor\dfrac{r-2k}{2012}\right\rfloor$

$\Leftrightarrow \left\lfloor\dfrac{r-k}{2011}\right\rfloor=\left\lfloor\dfrac{r-2k}{2012}\right\rfloor=0$

Suy ra: $r-2k\ge 0 \Rightarrow 2k\le r\le 2009\Rightarrow 0\le k\le 1004$

Vậy có $1005$ giá trị của k (từ $0$ đến $1004$). Tương ứng với mỗi giá trị $k$ thì $r$ nhận các giá trị từ $2k$ đến $2009$, suy ra có $2009-2k+1=2010-2k$ giá trị của $r$ tương ứng với mỗi giá trị của $k$

Như vậy số nghiệm tự nhiên của hệ trên là: $\sum\limits_{k=0}^{1004} (2010-2k)=1\;011\;030$ (em tính lại xem đúng không nhé!)

#8
hxthanh

hxthanh

    Tín đồ $\sum$

  • Hiệp sỹ
  • 3915 Bài viết
Bài 3: Giải phương trình:

$\left\lfloor-x^2+3x\right\rfloor=\left\lfloor x^2+\dfrac{1}{2}\right\rfloor$

Lời giải:
Đặt giá trị phần nguyên của 2 vế là $m$, ta có:
$m=\left\lfloor x^2+\dfrac{1}{2}\right\rfloor \ge \left\lfloor \dfrac{1}{2}\right\rfloor=0\Rightarrow m\ge 0$
Suy ra:
$-x^2+3x\ge 0 \Rightarrow 0\le x \le 3$.
Mặt khác: Theo bất đẳng thức AM-GM thì $x(3-x)\le (\dfrac{x+3-x}{2})^2=\dfrac{9}{4}<3$
Do đó: $m\in \{0,1,2\}$
Từ đây ta có 3 hệ bất phương trình tương ứng với 3 giá trị của $m$ là:

$(I)\;\;\;\begin{cases}0\le -x^2+3x <1 \\ 0\le x^2+\dfrac{1}{2}<1\end{cases}\;\;\;(II)\;\;\;\begin{cases}1\le -x^2+3x <2 \\ 1\le x^2+\dfrac{1}{2}<2\end{cases}\;\;\;\text{và}\;\;\;(III)\;\;\;\begin{cases}2\le -x^2+3x <3 \\ 2\le x^2+\dfrac{1}{2}<3\end{cases}$

Nghiệm của (I) là $0\le x <\dfrac{3-\sqrt 5}{2}$

Nghiệm của (II) là $\dfrac{\sqrt2}{2}\le x<1$

Nghiệm của (III) là $\sqrt{\dfrac{3}{2}}\le x<\sqrt{\dfrac{5}{2}}$

(Đề nghị bạn đọc tự giải và kiểm tra)

Nghiệm của phương trình đã cho là hợp của 3 khoảng trên $x\in \left[0,\dfrac{3-\sqrt 5}{2}\right)\cup\left[\dfrac{\sqrt2}{2},1\right)\cup\left[\sqrt{\dfrac{3}{2}},\sqrt{\dfrac{5}{2}}\right)$

#9
hxthanh

hxthanh

    Tín đồ $\sum$

  • Hiệp sỹ
  • 3915 Bài viết
Bài 4: (Rất đơn giản)
Giải phương trình: $\lfloor x \rfloor^2-\lfloor x \rfloor-2=0$

Ta có: $\lfloor x \rfloor^2-\lfloor x \rfloor-2=\left(\lfloor x\rfloor +1\right)\left(\lfloor x\rfloor -2\right)=0$
Từ đó suy ra:
hoặc $\lfloor x \rfloor=-1\Leftrightarrow -1\le x<0$
hoặc $\lfloor x \rfloor=2\Leftrightarrow 2\le x<3$
--------
HẾT!

#10
hxthanh

hxthanh

    Tín đồ $\sum$

  • Hiệp sỹ
  • 3915 Bài viết
Một số bài tập tự luyện trong chuyên đề phần nguyên

$\boxed{Bt1.6}$
Cho $A=\sqrt{4n^2+n},\; n\in\mathbb{N}$. Chứng minh rằng: $\{A\}\le \dfrac{1}{4}$

(Romania - 2003)


$\boxed{Bt1.7}$
Chứng minh rằng: $\left\lfloor 5x \right\rfloor + \left\lfloor 5y \right\rfloor \ge \left\lfloor 3x+y \right\rfloor + \left\lfloor x+3y \right\rfloor+ \left\lfloor x \right\rfloor + \left\lfloor y \right\rfloor$
Từ kết quả đó chứng minh $(5m)!(5n)!$ chia hết cho $m!n!(3m+n)!(3n+m)!$

(USAMO-1975)


$\boxed{Bt1.8}$
Tìm số tự nhiên $n$ nhỏ nhất sao cho $n!$ tận cùng bằng $290$ chữ số $0$

(HMMT-2003)


$\boxed{Bt1.10}$

Tính tổng: $S=\sum\limits_{k=0}^{2009}\left(\left\lfloor \dfrac{3^k+2010}{3^{k+1}} \right\rfloor + \left\lfloor \dfrac{2010-3^k}{3^{k+1}} \right\rfloor \right)$

$\boxed{Bt1.11}$
Chứng minh rằng: $\sum\limits_{k=0}^{+\infty} \left\lfloor \dfrac{x+2^k}{2^{k+1}}\right\rfloor = \left\lfloor x \right\rfloor$

$\boxed{Bt2.4}$

Tính tổng $S_n=\sum\limits_{k=1}^n \left\lfloor \sqrt{k}+ \dfrac{1}{2} \right\rfloor$

$\boxed{Bt2.5}$

Tính tổng $S_n=\sum\limits_{k=1}^n \left\lfloor 2\sqrt{k} \right\rfloor$

$\boxed{Bt2.6}$
Cho dãy số $\{U_n\}_1^\infty \; :\;\{1,2,2,2,3,3,3,3,3,4,4,4,4,4,4,4,5,...\}$
Được xác định bởi quy luật: $1$ số $1$; $3$ số $2$; $5$ số $3$;...;$2k-1$ số $k$;...
Tìm số hạng tổng quát của dãy số trên.

$\boxed{Bt2.11}$

Tính: $S=\sum\limits_{k=1}^n \left\lfloor \dfrac{k^2-3k+2}{5} \right\rfloor$

$\boxed{Bt2.12}$

Tính $S=\sum\limits_{k=1}^{n-1}\left\{ \dfrac{km}{n} \right\}$, với $m,n \in\mathbb{N}^*;\;n\ge 2$

(JMO-1995)

$\boxed{Bt2.13}$
Cho $\lambda$ là một số vô tỷ dương, $n$ là một số nguyên dương. Chứng minh rằng:

$\sum\limits_{k=1}^n \left\lfloor k\lambda \right\rfloor + \sum\limits_{k=1}^{\left\lfloor n\lambda \right\rfloor} \left\lfloor \dfrac{k}{\lambda} \right\rfloor = n\left\lfloor n\lambda \right\rfloor$

$\boxed{Bt2.14}$

Tính $S=\sum\limits_{k=1}^{\frac{n(n+1)}{2}} \left\lfloor \dfrac{\sqrt{8k+1}-1}{2} \right\rfloor$

$\boxed{Bt2.18}$
Cho $m,n$ là các số nguyên dương.
Tính $S=\sum\limits_{k=1}^n \left\lfloor \dfrac{m}{k} \right\rfloor$

$\boxed{Bt2.19}$
Cho $p$ là số nguyên tố lẻ, $q$ là số nguyên không chia hết cho $p$.
Chứng minh rằng:
$\sum\limits_{k=1}^{p-1} \left\lfloor (-1)^k k^2\dfrac{q}{p}\right\rfloor = \dfrac{(p-1)(q-1)}{2}$

$\boxed{Bt2.20}$
Cho $p$ là số nguyên tố lẻ,
Chứng minh rằng:
$\sum\limits_{k=1}^{p-1} \dfrac{k^p-k}{p} \equiv \dfrac{p+1}{2} \;\pmod{p}$

$\boxed{Bt2.21}$
Cho $p$ và $q$ là 2 số lẻ,
Tính giá trị biểu thức:

$A=\sum\limits_{k=1}^{\frac{p-1}{2}} \left\lfloor \dfrac{kq}{p} \right\rfloor + \sum\limits_{k=1}^{\frac{q-1}{2}} \left\lfloor \dfrac{kp}{q} \right\rfloor$

$\boxed{Bt2.22}$
Cho số nguyên $n\ge 2$

Tính: $S=\sum\limits_{m=1}^{ \left\lfloor \frac{n}{2} \right\rfloor } \sum\limits_{k=1}^{n+1-2m} \left\lfloor \dfrac{n-m}{k+m-1} \right\rfloor$
__________________________________

#11
E. Galois

E. Galois

    Chú lùn thứ 8

  • Quản lý Toán Phổ thông
  • 3861 Bài viết
Đề nghị anh Thanh và Toàn sớm hoàn thành chuyên đề này để đăng lên trang chủ

1) Xem cách đăng bài tại đây
2) Học gõ công thức toán tại: http://diendantoanho...oạn-thảo-latex/
3) Xin đừng đặt tiêu đề gây nhiễu: "Một bài hay", "... đây", "giúp tớ với", "cần gấp", ...
4) Ghé thăm tôi tại 
http://Chúlùnthứ8.vn

5) Xin đừng hỏi bài hay nhờ tôi giải toán. Tôi cực gà.


#12
tieulyly1995

tieulyly1995

    Sĩ quan

  • Thành viên
  • 435 Bài viết

Một số bài tập tự luyện trong chuyên đề phần nguyên

$\boxed{Bt1.7}$
Chứng minh rằng: $\left\lfloor 5x \right\rfloor + \left\lfloor 5y \right\rfloor \ge \left\lfloor 3x+y \right\rfloor + \left\lfloor x+3y \right\rfloor+ \left\lfloor x \right\rfloor + \left\lfloor y \right\rfloor$
Từ kết quả đó chứng minh $(5m)!(5n)!$ chia hết cho $m!n!(3m+n)!(3n+m)!$

(USAMO-1975)

__________________________________

Bài này thầy ghi nhầm đề rồi, trong phần tự luyện của thầy là thế này cơ mà : :closedeyes:
CMR :
$\left\lfloor 5x \right\rfloor + \left\lfloor 5y \right\rfloor \ge \left\lfloor 3x+y \right\rfloor + \left\lfloor x+3y \right\rfloor$
Từ kết quả đó chứng minh $(5m)!(5n)!$ chia hết cho $m!n!(3m+n)!(3n+m)!$

Giải:
Đặt $x=a+u$, $y=b+u$ ( a, b là các số nguyên không âm, $0\leq u,v< 1$)
$BĐT\Leftrightarrow a+b+\left \lfloor 5u \right \rfloor+\left \lfloor 5v \right \rfloor\geq \left \lfloor 3u+v \right \rfloor+\left \lfloor 3v+u \right \rfloor$
Ta sẽ CM bất đẳng thức mạnh hơn :
$\left \lfloor 5u \right \rfloor+\left \lfloor 5v \right \rfloor\geq \left \lfloor 3u+v \right \rfloor+\left \lfloor 3v+u \right \rfloor$ (*)
vì u,v có vai trò như nhau . Không mất tình tổng quát, ta giả sử $u\geq v$
$\Rightarrow \left \lfloor 5u \right \rfloor\geq \left \lfloor 3u+v \right \rfloor$
Nếu $u\leq 2v$ thì $\left \lfloor 5v \right \rfloor\geq \left \lfloor 3v+u \right \rfloor$: $\Rightarrow$ đpcm
Với $u> 2v$ :
Đặt $5u=a^{'}+b^{'}, 5v=c^{'}+d^{'}$ ( $a^{'}, c^{'}$ là các số nguyên không âm , $0\leq b^{'},d^{'}< 1$)
$(*) \Leftrightarrow a^{'}+c^{'}= \left \lfloor \frac{3a^{'}+c^{'}+3b^{'}+d^{'}}{5} \right \rfloor+\left \lfloor \frac{3c^{'}+a^{'}+3d^{'}+b^{'}}{5} \right \rfloor$ (**)
Vì $1> u> 2v \Rightarrow 5> 5u> 10v \Rightarrow 5> a^{'}+b^{'}> 2c^{'}+2d^{'}\Rightarrow 5> a^{'} \Rightarrow 4\geq a^{'}$
Mà:
$a^{'}+b^{'}> 2c^{'}+2d^{'}\Rightarrow a^{'}\geq 2c^{'}$
( vì nếu$a^{'}< 2c^{'}\Rightarrow a^{'}\leq 2c^{'}-1\Rightarrow a^{'}+1-2c^{'}\leq 0, a^{'}+b^{'}-2c^{'}< 0$)
Do đó:
$4\geq a^{'}\geq 2c^{'}$
Kiểm tra (**) đối với 9 trường hợp
( chú ý là $3b^{'}+d^{'}< 4, 3d^{'}+b^{'}< 4$):
$a^{'}= 4, c^{'}= 2,1,0$
$a^{'}= 3, c^{'}= 1,0$
$a^{'}= 2, c^{'}= 1,0$
$a^{'}= 1, c^{'}= 0$
$a^{'}=0, c^{'}= 0$
đúng $\Rightarrow đpcm$
Áp dụng:
Theo định lý Legendre( để biết thêm chi tiết ,các bạn vào chuyên đề phần nguyên của thầy hxthanh), ta chỉ cần CM
$\left \lfloor \frac{5m}{r} \right \rfloor+\left \lfloor \frac{5n}{r} \right \rfloor\geq \left \lfloor \frac{m}{r} \right \rfloor+\left \lfloor \frac{n}{r} \right \rfloor+\left \lfloor \frac{3m+n}{r} \right \rfloor+\left \lfloor \frac{3n+m}{r} \right \rfloor$, với mọi $r\geq 2$ (^^)
đặt $m=rm^{'}+x, n=rn^{'}+y$
(trong đó $0\leq x,y< r$ ; r,$m^{'}, n^{'}$ nguyên )
Khi đó
(^^) trở thành :
$\left \lfloor \frac{5x}{r} \right \rfloor+\left \lfloor \frac{5y}{r} \right \rfloor\geq \left \lfloor \frac{3x+y}{r} \right \rfloor+\left \lfloor \frac{3y+x}{r} \right \rfloor$(cmt)
THE END. :icon6:
p/s : Bài có gì sai sót, mong mọi người bổ sung thêm :P

Bài viết đã được chỉnh sửa nội dung bởi perfectstrong: 12-02-2012 - 20:49


#13
hxthanh

hxthanh

    Tín đồ $\sum$

  • Hiệp sỹ
  • 3915 Bài viết

Một số bài tập tự luyện trong chuyên đề phần nguyên

$\boxed{Bt1.6}$
Cho $A=\sqrt{4n^2+n},\; n\in\mathbb{N}$. Chứng minh rằng: $\{A\}\le \dfrac{1}{4}$

(Romania - 2003)


Lời giải:

Ta có: $4n^2 \le 4n^2+n < 4n^2+4n+1$
Suy ra: $\left\lfloor A \right\rfloor = 2n \Rightarrow \left\{ A \right\} = A - \left\lfloor A \right\rfloor = \sqrt{4n^2+n} - 2n$
Bất đẳng thức cần chứng minh tương đương với:
$\sqrt{4n^2+n} - 2n \le \dfrac{1}{4};\;(n\in \mathbb{N})$
$\Leftrightarrow 4\sqrt{4n^2+n} \le 8n+1$
$\Leftrightarrow 16(4n^2+n) \le 64n^2+16n+1 $
$\Leftrightarrow 0 \le 1 \Leftrightarrow \text{ true}$

#14
hxthanh

hxthanh

    Tín đồ $\sum$

  • Hiệp sỹ
  • 3915 Bài viết
Hai bài phần nguyên trong kỳ thi của ĐHKHTN các năm trước
$\begin{array}{| |}
\hline
&\boxed{\text{Bài 1}}&\\
&\text{Chứng minh rằng với mọi } n \text{ tự nhiên thì:}&\\
&\left\lfloor\sqrt[3]{72n+1}\right\rfloor=\left\lfloor\sqrt[3]{9n}\right\rfloor+\left\lfloor\sqrt[3]{9n+1}\right\rfloor=\left\lfloor\sqrt[3]{72n+7}\right\rfloor&\\
&&&\\
\hline
\end{array}$

$\begin{array}{| |}
\hline
&\boxed{\text{Bài 2}}&\\
&\text{Chứng minh rằng với mọi số nguyên dương } n \text{ thì:}&\\
&A=n+\left\lfloor\sqrt[3]{n-\dfrac{1}{27}}+\dfrac{1}{3}\right\rfloor^2&\\
&\text{không thể là một lập phương đúng!}&
&&&\\
\hline
\end{array}$

#15
hxthanh

hxthanh

    Tín đồ $\sum$

  • Hiệp sỹ
  • 3915 Bài viết

$\begin{array}{| |}
\hline
&\boxed{\text{Bài 2}}&\\
&\text{Chứng minh rằng với mọi số nguyên dương } n \text{ thì:}&\\
&A=n+\left\lfloor\sqrt[3]{n-\dfrac{1}{27}}+\dfrac{1}{3}\right\rfloor^2&\\
&\text{không thể là một lập phương đúng!}&
&&&\\
\hline
\end{array}$

Lời giải:
Đặt $m=\left\lfloor\sqrt[3]{n-\dfrac{1}{27}}+\dfrac{1}{3}\right\rfloor=\left\lfloor\dfrac{\sqrt[3]{27n-1}+1}{3}\right\rfloor$
Dễ thấy $m \ge 1, \quad \forall n \ge 1$
Ta có:
$\begin{array}{lrcccl}
& m &\le& \dfrac{\sqrt[3]{27n-1}+1}{3} &<& m+1\\
\Leftrightarrow & 3m-1 &\le& \sqrt[3]{27n-1}&<& 3m+2\\
\Leftrightarrow & 27m^3-27m^2+9m-1 &\le& 27n-1&<& 27m^3+54m^2+36m+8\\
\Leftrightarrow & m^3-m^2+\dfrac{m}{3} &\le& n&<& m^3+2m^2+\dfrac{4m+1}{3}\\
\Leftrightarrow & m^3+\dfrac{m}{3} &\le& n+m^2&<& m^3+3m^2+\dfrac{4m+1}{3}\\
\Rightarrow & m^3 &<& A&<& (m+1)^3\\
\end{array}$
Do $m^3$ và $(m+1)^3$ là $2$ số lập phương liên tiếp nên $A$ không thể là số lập phương
$\blacksquare$

#16
hxthanh

hxthanh

    Tín đồ $\sum$

  • Hiệp sỹ
  • 3915 Bài viết
Để tính toán các tổng phần nguyên hữu tỉ, ta có một BỔ ĐỀ khá quan trọng sau:
$$\begin{array}{| |}
\hline
\boxed{\text{ BỔ ĐỀ I }}\\
\text{Với mọi số nguyên dương } m \text{ và } n \text{ ta có: }\\
\boxed{\sum\limits_{k=0}^n \left\lfloor\dfrac{k}{m}\right\rfloor=\left(n+1-\dfrac{m}{2}\right)\left\lfloor\dfrac{n}{m}\right\rfloor-\dfrac{m}{2}\left\lfloor\dfrac{n}{m}\right\rfloor^2}\\
\hline
\end{array}$$
Chứng minh:
Với $m=1$: Bổ đề trở thành công thức quen thuộc $\sum\limits_{k=0}^n k =\dfrac{n(n+1)}{2}$.
Xét với $m \ge 2$

Ta sẽ nhóm các số hạng theo số thứ tự đồng dư modul $m$ bằng cách đặt
$k=pm+r,\quad (0 \le r \le m-1)$
Khi đó $0 \le p \le \left\lfloor\dfrac{n}{m}\right\rfloor-1$, phần "dôi ra" sẽ chạy từ $k=m\left\lfloor\dfrac{n}{m}\right\rfloor $ đến $k=n$


Ta có:
$\sum\limits_{k=0}^n \left\lfloor\dfrac{k}{m}\right\rfloor=\sum\limits_{p=0}^{\left\lfloor\frac{n}{m}\right\rfloor-1}\sum\limits_{r=0}^{m-1} \left\lfloor\dfrac{pm+r}{m}\right\rfloor+\sum\limits_{k=m\left\lfloor\frac{n}{m}\right\rfloor}^n \left\lfloor\dfrac{k}{m}\right\rfloor=\sum\limits_{p=0}^{\left\lfloor\frac{n}{m}\right\rfloor-1} mp + \left(n+1-m\left\lfloor\dfrac{n}{m}\right\rfloor\right)\left\lfloor\dfrac{n}{m}\right\rfloor$
$\qquad\qquad = m\left(\dfrac{1}{2}\left(\left\lfloor\dfrac{n}{m}\right\rfloor-1\right)\left\lfloor\dfrac{n}{m}\right\rfloor\right)+(n+1)\left\lfloor\dfrac{n}{m}\right\rfloor-m\left\lfloor\dfrac{n}{m}\right\rfloor^2$
$\qquad\qquad=\left(n+1-\dfrac{m}{2}\right)\left\lfloor\dfrac{n}{m}\right\rfloor-\dfrac{m}{2}\left\lfloor\dfrac{n}{m}\right\rfloor^2$
Vậy ta có điều phải chứng minh.
$\square$
Sau đây ta sẽ áp dụng BỔ ĐỀ này để giải quyết Bài toán $\boxed{Bt2.11}$

#17
hxthanh

hxthanh

    Tín đồ $\sum$

  • Hiệp sỹ
  • 3915 Bài viết

$\boxed{Bt2.11}$

Tính: $S=\sum\limits_{k=1}^n \left\lfloor \dfrac{k^2-3k+2}{5} \right\rfloor$

- Bài này nhìn gọn gàng đẹp đẽ là thế, nhưng tính toán lại chẳng đơn giản chút nào!
- Việc đầu tiên là ta cần phải "hạ bậc" đối với tử thức của phân thức trong dấu phần nguyên, và dĩ nhiên ta phải xét theo modul $5$
- Đặt $k=5p+r$ với $0 \le r \le 4$ và để cho tiện, khỏi phải xét riêng lẻ $5$ trường hợp ta viết gộp $r=(0,1,2,3,4)$
hay là $k=5p+(0,1,2,3,4)$
Thay vào biểu thức trong dấu phần nguyên ta được:

$\quad\left\lfloor \dfrac{k^2-3k+2}{5} \right\rfloor=\left\lfloor \dfrac{\Big(5p+(0,1,2,3,4)\Big)^2-3\Big(5p+(0,1,2,3,4)\Big)+2}{5} \right\rfloor=$

$=\left\lfloor \dfrac{\Big(25p^2+10p(0,1,2,3,4)+(0,1,4,9,16)\Big)-\Big(15p+(0,3,6,9,12)\Big)+(2,2,2,2,2)}{5} \right\rfloor$
$=5p^2+2pr-3p+\left\lfloor\dfrac{(2,0,0,2,6)}{5}\right\rfloor$
$=5p^2+2pr-3p+(0,0,0,0,1)$
$=5p^2+2pr-3p+\left\lfloor\dfrac{r+1}{5}\right\rfloor$
$=5p^2-4p+2pr+\left\lfloor\dfrac{k+1}{5}\right\rfloor$
Từ đó suy ra:
$S=\sum\limits_{k=1}^n \left\lfloor \dfrac{k^2-3k+2}{5} \right\rfloor=\sum\limits_{1\le k=5p+r \le n} \left(5p^2-4p+2pr+\left\lfloor\dfrac{k+1}{5}\right\rfloor\right)$
$=\sum\limits_{1\le k=5p+r \le n} \left(5p^2-4p+2pr\right)+\sum\limits_{k=1}^n\left\lfloor\dfrac{k+1}{5}\right\rfloor$
$\begin{array}{}
&& && \\
=&\sum\limits_{p=0}^{\left\lfloor\frac{n}{5}\right\rfloor-1}\sum\limits_{r=0}^{4}\left(5p^2-4p+2pr\right)+\sum\limits_{p=\left\lfloor\frac{n}{5}\right\rfloor} \sum\limits_{r=0}^{n-5\left\lfloor\frac{n}{5}\right\rfloor}\left(5p^2-4p+2pr\right)+\left\lfloor\dfrac{n+1}{5}\right\rfloor&+\sum\limits_{k=0}^n \left\lfloor\dfrac{k}{5}\right\rfloor\end{array}$

$=\sum\limits_{p=0}^{\left\lfloor\frac{n}{5}\right\rfloor-1}\left(25p^2\right)+\sum\limits_{r=0}^{n-5\left\lfloor\frac{n}{5}\right\rfloor}\left(5\left\lfloor\dfrac{n}{5}\right\rfloor^2-4\left\lfloor\dfrac{n}{5}\right\rfloor+2\left\lfloor\dfrac{n}{5}\right\rfloor r\right)+\left\lfloor\dfrac{n+1}{5}\right\rfloor+\sum\limits_{k=0}^n \left\lfloor\dfrac{k}{5}\right\rfloor$

$=\dfrac{25}{6}\left\lfloor\dfrac{n}{5}\right\rfloor\left(\left\lfloor\dfrac{n}{5}\right\rfloor-1\right)\left(2\left\lfloor\dfrac{n}{5}\right\rfloor-1\right)+\left(n+1-5\left\lfloor\dfrac{n}{5}\right\rfloor\right)\left(5\left\lfloor\dfrac{n}{5}\right\rfloor^2-4\left\lfloor\dfrac{n}{5}\right\rfloor\right)+ \qquad+2\left\lfloor\dfrac{n}{5}\right\rfloor\dfrac{1}{2}\left(n-5\left\lfloor\dfrac{n}{5}\right\rfloor\right)\left(n+1-5\left\lfloor\dfrac{n}{5}\right\rfloor\right)+\left\lfloor\dfrac{n+1}{5}\right\rfloor+\left(\left(n+1-\dfrac{5}{2}\right)\left\lfloor\dfrac{n}{5}\right\rfloor-\dfrac{5}{2}\left\lfloor\dfrac{n}{5}\right\rfloor^2\right)\quad\text{(Theo Bổ đề I)}$

Khai triển ra rồi rút gọn lại thì được:

$\boxed{S=\sum\limits_{k=1}^n \left\lfloor \dfrac{k^2-3k+2}{5} \right\rfloor=\left\lfloor\dfrac{n}{5}\right\rfloor\left(n^2-2n-\dfrac{4}{3}+5\left\lfloor\dfrac{n}{5}\right\rfloor-5n\left\lfloor\dfrac{n}{5}\right\rfloor+\dfrac{25}{3}\left\lfloor\dfrac{n}{5}\right\rfloor^2\right)+\left\lfloor\dfrac{n+1}{5}\right\rfloor}$
_____________________________________________________________
p/s: Nhìn chóng mặt quá nhỉ :D
So sánh một số giá trị của Vế Trái
và một số giá trị tính được ở Vế Phải

#18
hxthanh

hxthanh

    Tín đồ $\sum$

  • Hiệp sỹ
  • 3915 Bài viết

$\boxed{Bt2.4}$

Tính tổng $S_n=\sum\limits_{k=1}^n \left\lfloor \sqrt{k}+ \dfrac{1}{2} \right\rfloor$

Lời giải:
(Có nhiều cách để tính được tổng này, ở đây ta làm cách thủ công nhất đó là chia đoạn)

Đặt $m=\left\lfloor \sqrt{n}+ \dfrac{1}{2} \right\rfloor$

Suy ra: $m \le \sqrt{n}+ \dfrac{1}{2} < m+1$ (Thực tế không có dấu bằng)

$\Rightarrow \left(m-\dfrac{1}{2}\right) \le \sqrt{n} < \left(m+\dfrac{1}{2}\right)$

$\Rightarrow \left(m^2-m+\dfrac{1}{4}\right) \le n < \left(m^2+m+\dfrac{1}{4}\right)$

$\Rightarrow \left(m^2-m+1\right) \le n < \left(m^2+m+1\right)$

Do đó ta chia đoạn $[1,n]$ thành các đoạn $[i^2-i+1,i^2+i]\;1\le i \le m-1$ đoạn cuối là $[m^2-m+1,n]$

Trên mỗi đoạn thứ $i$, tương ứng $i^2-i+1 \le k \le i^2+i$ thì $\left\lfloor \sqrt{k}+ \dfrac{1}{2} \right\rfloor=i$

Trên đoạn cuối cùng, tương ứng $m^2-m+1\le k \le n$ thì $\left\lfloor \sqrt{k}+ \dfrac{1}{2} \right\rfloor=m$
Suy ra:
$S_n=\sum\limits_{k=1}^n \left\lfloor \sqrt{k}+ \dfrac{1}{2} \right\rfloor= \sum\limits_{i=1}^{m-1}\sum\limits_{k=i^2-i+1}^{i^2+i} i + \sum\limits_{k=m^2-m+1}^n m$
$S_n=\sum\limits_{i=1}^{m-1} (2i^2) + m(n+m-m^2)$
$S_n=\dfrac{2m(m-1)(2m-1)}{6} + m(n+m-m^2)$

$\boxed{S_n=\sum\limits_{k=1}^n \left\lfloor \sqrt{k}+ \dfrac{1}{2} \right\rfloor=mn-\dfrac{m(m-1)(m+1)}{3},\quad \text{với } m=\left\lfloor\sqrt{n}+ \dfrac{1}{2}\right\rfloor}$

#19
hxthanh

hxthanh

    Tín đồ $\sum$

  • Hiệp sỹ
  • 3915 Bài viết
Một bài toán tương tự đó là bài KMO 1997
Tính $S=\sum\limits_{k=1}^n \left\lfloor\sqrt{k}\right\rfloor$

Giải tương tự như $\boxed{Bt2.4}$ ta có kết quả

$S=\sum\limits_{k=1}^n \left\lfloor\sqrt{k}\right\rfloor = np-\dfrac{p(p-1)(2p+5)}{6};\quad\text{ với }p=\left\lfloor\sqrt{n}\right\rfloor$

Sử dụng kết quả này và kết quả của $\boxed{Bt2.4}$ với định lý Hermite ta dễ dàng xử lý bài

$\boxed{Bt2.5}$

Tính tổng $S_n=\sum\limits_{k=1}^n \left\lfloor 2\sqrt{k} \right\rfloor$

Áp dụng định lý Hermite ta có $\left\lfloor\sqrt{k}\right\rfloor+\left\lfloor\sqrt{k}+\dfrac{1}{2}\right\rfloor = \left\lfloor 2\sqrt{k}\right\rfloor$
Từ đó dễ dàng tính được kết quả!

#20
hxthanh

hxthanh

    Tín đồ $\sum$

  • Hiệp sỹ
  • 3915 Bài viết

$\boxed{Bt2.6}$
Cho dãy số $\{U_n\}_1^\infty \; :\;\{1,2,2,2,3,3,3,3,3,4,4,4,4,4,4,4,5,...\}$
Được xác định bởi quy luật: $1$ số $1$; $3$ số $2$; $5$ số $3$;...;$2k-1$ số $k$;...
Tìm số hạng tổng quát của dãy số trên.

(Bài này dễ mà :P )
Lời giải:
Xét $U_n$ trong nhóm có giá trị $k$, (nhóm này có $2k-1$ số hạng)
Số các số hạng phía trước nó có $(k-1)$ nhóm, số phần tử của mỗi nhóm là các số lẻ liên tiếp, do đó:
$\Rightarrow \sum\limits_{i=1}^{k-1}(2i-1)<n<\sum\limits_{i=1}^{k-1}(2i-1)+2k$

$\Rightarrow (k-1)^2<n<k^2+1$

$\Rightarrow (k-1)^2+1 \le n < k^2+1$

$\Rightarrow \sqrt{n-1}< k \le \sqrt{n-1}+1$

$\Rightarrow k=1+\left\lfloor\sqrt{n-1}\right\rfloor$

Vậy số hạng tổng quát của dãy là $U_n=1+\left\lfloor\sqrt{n-1}\right\rfloor,\qquad\forall n\ge 1$
$\blacksquare$




0 người đang xem chủ đề

0 thành viên, 0 khách, 0 thành viên ẩn danh